What is the cost of riding a taxi for 25 km if the taxi has a base fee of $5 and an additional $2.60 per kilometre? Use the equation C=2.6k+5, where C is the cost and k is the number of kilometres travelled.

Answers

Answer 1
70 km because 2.5*25 is 65, and 65 + 5 is 70

Related Questions

what 50 precent of 7

Answers

Answer:

3.5

7 divided by 50=3.5

3.5x2=7

Step-by-step explanation:

A truck is carrying mango juice and grapefruit juice bottles. The ratio of mango juice to grapefruit juice is 3:4 How many mango juice bottles are there if there are 24 grapefruit juice bottles? Write a ratio and use x as your variable.​

Answers

Answer:

18

Step-by-step explanation:

3/4 of 24 is 18.

Another way of writing it is 75% of 24 = 18

Sam put a 12-inch ruler into a cup. If 3 inches of the ruler were sticking out of the top of the cup, how deep was the cup?

Answers

Answer:

9 inches deep

Step-by-step explanation:

12 inches - 3 inches = 9 inches

Solve the proportion 10/p+2 =4/3

Answers

Answer:

p=11/2

Step-by-step explanation:

there are algebra calculators

What is the coefficient in the variable term 7x?

Answers

Answer:

7

Step-by-step explanation:

the coefficient is the number that is multiplied by x which in the case of 7x is 7

Hope that helps :)

Point B has coordinates ​(4,1​). The​ x-coordinate of point A is -8. The distance between point A and point B is 13 units. What are the possible coordinates of point​ A?

Answers

Answer:

The possible coordinates of point A are (-8,-4) or (-8,6)

Step-by-step explanation:

The distance between two points is given by the formula

[tex]d = \sqrt{(x_{2}-x_{1})^{2} + (y_{2}-y_{1})^{2} }[/tex]

For point A [tex](x_{1} , y_{1})[/tex]; [tex]x_{1}= -8[/tex] and [tex]y_{1}[/tex] is unknown

For point B [tex](x_{2} , y_{2})[/tex]; (4,1) i.e [tex]x_{2}=4[/tex] and [tex]y_{2}=1[/tex]

and d = 13

Putting the values into the equation,

[tex]13= \sqrt{[(4-(-8)]^{2} + (1-y_{1})^{2} }\\13= \sqrt{[(4+8)]^{2} + (1-y_{1})^{2} }\\13= \sqrt{12^{2} + (1-y_{1})^{2} }\\13^{2} = 12^{2} + (1-y_{1})^{2} \\169 = 144 +(1-y_{1})^{2}\\169-144 = (1-y_{1})^{2}\\25 = 1 -2y_{1} +y_{1}^{2} \\y_{1}^{2} -2y_{1}+1-25 =0\\y_{1}^{2} -2y_{1}-24 =0\\y_{1}^{2} -6y_{1} + 4y_{1} -24 =0 \\y_{1}(y_{1}-6) +4(y_{1} -6) = 0\\(y_{1}+4)(y_{1}-6) =0\\(y_{1}+4)=0 or (y_{1}-6) =0[/tex]

[tex]y_{1}+4= 0[/tex] or [tex]y_{1}-6 =0[/tex]

[tex]y_{1} =-4[/tex] or [tex]y_{1} = 6[/tex]

Hence, the possible coordinates of point A are (-8,-4) or (-8,6)

Need help on this question.

Answers

Answer:

The constant difference is 8

Step-by-step explanation:

-6 + 8 = 2

10 + 8 = 18


I need help asap please

Answers

Answer:

x = - 69

Step-by-step explanation:

if its wrong i can help u again

I’m pretty the answer would be -69

Almonds cost $12 per 3 pounds. How many pounds can I buy with $20?

Answers

Answer:

6 pounds

Step-by-step explanation:

Answer:

5 pounds

Step-by-step explanation:

12 divided by 3 = 4

20 divided by 4 = 5

simplify the rational expression: x-a/7x + 5x-5a/7x

Answers

Answer:6x - (6xa)/(7)

Step-by-step explanation:

Which expression is equivalento = 92 +12) +41 +62
- - -
BT2 = 10
13
32=5
131 - IS
ASAP

Answers

Answer:

i dont understand

Step-by-step explanation:


Conor has $200 available to buy software
packages to run updates on his employee
computers. Each software package has a
price of $12, and Conor will pay a sales tax
of 8% of the total price of the software.
What is the maximum number of software?

packages Conor can buy?

Answers

Don’t go to that link it is a virus it’s a bot doing that do not go to that link

WILL GIVE BRAINLIEST!
d(n)=3(−2) ^(n-1)
d(n)=3(−2)
n−1

What is the 4th term in the sequence?

Answers

Answer:

-24

Step-by-step explanation:

B-30.45=95.28 what is the answer

Answers

Answer:

[tex]B = 125.73[/tex]

Step-by-step explanation:

Step 1:  Add 30.45 to both sides

[tex]B - 30.45=95.28[/tex]

[tex]B +(- 30.45 + 30.45)=(95.28 + 30.45)[/tex]

[tex]B = 125.73[/tex]

Answer:  [tex]B = 125.73[/tex]

Kenneth has 45 Pokemon cards. He gave 3/9 of his cards to his friend as a random act of kindness. How many cards did he give his friend? m​

Answers

Answer:

15 pokémon cards

Step-by-step explanation:

45*3 /9= 135/9 = 15

Or you can simplify

3/9=1/3

45*1/3=15

Answer:

15

Step-by-step explanation:

45/9=5 5*3=15

I divide 45 by 9 to see how much card 1/9 is

regan's website had 7 links on each of 9 pages. how many links did she have on her website? i need equation and solution!!

Answers

Answer:

63

Step-by-step explanation:

all you need to do is 9*7

If Evan has 3 Jars of Cookies and each jar holds 18 cookies how many cookies does Evan have?

Answers

Answer:54 cookies in total

Step-by-step explanation: 3x18=54

Answer: 54 cookies in all

Step-by-step explanation: Evan has 54 cookies because if you do 18x3 you get 54, or if you do 18+18+18 you get 54.

=

Jules buys a washing machine.
20% VAT is added to the price of the washing machine.
Jules then has to pay a total of £600.
What is the price Is the price of the washing machine with no VAT added

Answers

Answer:£500

Step-by-step explanation: 600 ÷ 1.05

Answer:

£500

Step-by-step explanation:

(2x + 4) + (-7 - 3b) + (5 - 8x)
)
Please help

Answers

Answer:

Step-by-step explanation:

First, open the brackets.

(2x + 4) + (-7 - 3b) + (5 - 8x)

= 2x + 4 -7 - 3b + 5 -8x

Simplify the like terms.

= 2x - 8x - 3b + 4 -7 + 5

= -6x - 3b + 2

6y - 3 - 2y = -3

(substitution)

Answers

Answer:

y = 0

Step-by-step explanation:

HELP I NEED HELP ASAP
HELP I NEED HELP ASAP
HELP I NEED HELP ASAP

Two numbers have a sum of 76 and a difference of 8. What is the larger number?

A. 34
B. 46
C. 68
D. 42

Answers

Answer:

46,42

( I hope this was helpful) >;D

Everyday fitness health club charges $18 per month, new members must pay a $60 fee. What is an equation to represent the relationship between cost and number of months

Answers

c = 18m + 60
if it’s slope intercept form you want

4 2/3 - 3 1/8
Leave your answer as a mixed fraction

Answers

Answer:

Here.is the answer.

Step-by-step explanation:

Hope it helps.....

Giving brainliest if answered correctly!

At Bakersfield High School, each senior class purchases a monument in the shape of a triangular prism to display the names of the entire senior class. The dimensions of the monument are shown below.

What is the total surface area of the monument?

660 square inches
630 square inches
690 square inches
600 square inches

Answers

Answer:

hello! you didnt put the dimensions. i would love to help but i dont have the info. you can comment the infor below!

A rectangular garden has a length that is 4 feet longer than its
width. The entire garden is surrounded by a 1-foot-wide decorative
brick pattern on the ground. Write an expression for the total area
of the garden and decorative brick pattern.

Answers

Answer:

(W + 4) = L

W = W

(W + 4) * W

Step-by-step explanation:

plz i got need help here​

Answers

Answer:

The parallelograms having perpendicular diagonals are square and rhombus so

e. b and d are true.

Hope it will help :)❤

Answer:

I think Rhombus

Step-by-step explanation:

‘z’ is subtracted from (-12) express the expression​

Answers

Answer:

Z + 12

Step-by-step explanation:

Z-(-12)

Z + 12

Double negative turns into positive

keith buys 6 pencils for 90p
how much does one pencil cost?
how much would five pencils cost
how much would eleven cost ?

Answers

15p per pencil for 6 pencils
75p for 5 pencils
1.65 for 11 pencils

Answer:

1 Pencil = 15p

5 Pencils = 75p

11 Pencils = 165p --> £1.65

Step-by-step explanation:

To find one pencil... 90 / 6 = 15

To find 5 pencils... 15 (worth 1 pencil) * 5 = 75

To find 11 pencils... 15 (worth 1 pencil) * 11 = 165 which is £1.65

Please help please I need to know this

Answers

Answer: X=19

Step-by-step explanation:

Add up everything and set it to 180 degrees so (5x+4)+(x-2)+(3x+7)=180

next 9x+9=180

then 9x=171

x=19

If you want to check by pluggin in value for x for all angles and it should equal 180 added up

Angle POS= 99

Angle SOR= 17

Angle ROQ= 64

99+17+64=180 degrees

Answer:

i have made it in above picture

hope it helps

1/3 HB=T solve for B​

Answers

Answer:

yes I agree B=3t/h

Step-by-step explanation:

hope this helps :)

Other Questions
Una fbrica produce 12 480 borradores cada semana. Estos artculos se venden empacados en cajas de una docena. Cuntas cajas de borradores saldrn a la venta cada semana? Why should a reader evaluate the arguments made in a speech?Question 2 options:To find out if the authors argument agrees with the readers viewsTo find out the order in which the arguments are madeTo know whether the evidence given by the author is credibleTo determine how to respond to the argument Marks: 1"Pepperoni pizza is my favoritefood." is an example of what kind ofsentence?o(a)Assertive(b)Imperative(CInterrogative(d)Exclamatory PLESSS PLESS PLESSSSSSS I BEG OF YOU TEACH ME HOW TO DO THISSSS I BEGG I WILL GIVE BRAINYLIST TO ANYONE WHO ANSWERS PLESS I CRY FOR YOU HELP AND BEGGG TOO Which equation will help you solve this problem?You have 16 books and 2 shelves to store them.If you split them equally between the 2 shelves, how many books will be on each shelf? Name THREE products in the food industry that are produced by biotechnology How does the author of "Solar-Powered Public Buildings? Not So Fast, California!" support the claim that an increase in solar power usage could disrupt power delivery to individuals and organizations? need help please. urgent!! What is the wavelength of this wave Whats your favorite book for branliest Ill go first mine secret garden!! HELPPPP NO ONE CAN ANSWER THIS QUESTION ITS DUE TODAY ITS WORTH ALOT OF POINTSS PLEASEEE question: what is your favorite song, my spotify playlists are starting to be dry as heck plz answer ASAP will get ten points Mom is a dominant purebred for big feet. Dad is a hybrid for foot size. What are their chances of big feet? Or little feet? _ out of _ or _ % Understanding Fractions as Division - Item 30582 A recipe calls for 12 cups of blueberries to make 7 jars of blueberry jam. What is true about the number of cups of blueberries in each jar? 7 Each jar has 12 cup of blueberries. Each jar has 1 of 7 cups of blueberrie Each jar has 12 cups of blueberries. Each jar has 1 cups of blueberries. . Which statement best expresses the central idea in the article? A. Scientists have discovered a way to prevent genetic defects in babies, but some believe genetic development should be left to nature B. Scientists have discovered a way to prevent genetic defects in babies, but some worry that the process could be used for great economic gain C. Scientists have discovered a way to prevent genetic defects in babies, but the method is against many families religions D. Scientists have discovered a way to prevent genetic defects in babies, but the process is still very risky Is this table linear or not? civil war questions like what ifs... or what would have happened if....?!?? help give thanks stars nb vjdbucvbdevohdbvhdqvhknabvjdnam quotient 2 divided by 2 3